Wie löst man die "EM-Wellengleichung" für das Feld der sich gleichmäßig bewegenden Ladung?

Kann man zeigen, dass das Feld einer gleichförmig bewegten Ladung, das nach dem Biot-Savart-Gesetz gegeben ist, gegeben ist durch...

E ( R , T ) = k Q ( 1 v 2 / C 2 ( 1 v 2 Sünde 2 θ / C 2 ) 3 / 2 ) ( R ^ γ 2 ( X v T ) 2 + j 2 + z 2 )

... die folgende Gleichung erfüllt (weit entfernt von der Ladung selbst)?

C 2 2 E = 2 T 2 E

Bearbeiten:

Ich weiß , dass eine gleichmäßige Bewegung keine Wellen erzeugt. Ich weiß auch , dass diese Gleichung nur für den leeren Raum weit entfernt von Ladungen und Strömen gilt.

Aber irgendwie muss diese Gleichung immer noch für das elektrische Feld gelten, das träge hinter der sich mit konstanter Geschwindigkeit bewegenden Ladung "geschleppt" wird.

Es ist wirklich schwierig für mich, diese Idee zu begreifen. Jegliche Referenzen wären sehr willkommen. Ich habe versucht, es direkt zu lösen, bin aber gescheitert.

Sie sollten Ihren Beitrag bearbeiten, um deutlich zu machen, dass Sie wissen, dass eine sich gleichmäßig bewegende Ladung keine Welle erzeugt, die Felder jedoch anscheinend diese Gleichung erfüllen müssen. Ich kenne die Antwort nicht (Up-Voting-Frage), aber ich stelle fest, dass Ihre Wellengleichung keinen Quellterm enthält ... vielleicht macht das einen Unterschied.
Ein Ansatz dazu wäre, zunächst zum Ruhesystem der Ladung zu gehen, in diesem System ist nämlich das elektrische Feld besonders einfach E = Q 4 π ϵ 0 R 2 , wo es einfach ist zu zeigen (da es keine Zeitableitungen gibt), dann in einen Rahmen zu transformieren, in dem sich die Ladung bewegt, und dies zu bemerken 2 T 2 + 1 C 2 2 unveränderlich ist, siehe zum Beispiel Purcell, Electricity and Magnetism .
Hallo @jim, ist es richtig zu sagen, dass die sogenannte "EM-Wellengleichung" offensichtlich nicht wellenartige Lösungen hat (wie ein sich inertial bewegendes Feld)? Warum heißt es überhaupt Wellengleichung?
@VitalyKorzhik Ich denke, weil die EM-Gleichung wellenartige Lösungen zulassen kann, wird sie als EM-Wellengleichung bezeichnet . Bei zeitunabhängigen Gleichungen wird kein besonderer Unterschied gemacht. Außerdem können Sie zeitabhängige Lösungen haben, die nicht sinusförmig sein müssen, die die Leute normalerweise als Welle betrachten.
@jim Gibt es einen offensichtlichen Grund, warum die Feldfunktion nicht zufriedenstellend ist v 2 2 E = 2 T 2 E (mit v anstatt C )? Weil es nicht beide gleichzeitig befriedigen kann. Das hat wahrscheinlich etwas mit der Lorentz-Transformation zu tun ...
Ja, die Lorentz-Transformation bedeutet das nur wann v = C ist die Wellengleichungsinvariante. Eine andere Möglichkeit, darüber nachzudenken, ist für v < C es wäre möglich, zu einem Frame zu gehen, wo v = 0 was bedeutet, dass Sie auf einen Rahmen anheben könnten, in dem Em-Wellen statisch wären. Die folgende Frage und Antwort könnte von Interesse sein: physical.stackexchange.com/questions/248499/…

Antworten (1)

Die Leute verstehen Ihre Frage nicht. Ich denke, Sie möchten, dass jemand explizit überprüft, ob die in Ihrem Spezialfall erzeugten Felder der allgemein gültigen Wellengleichung gehorchen (oder nicht) . Schließlich sieht das erzeugte Feld nicht wie eine Welle aus. Eine allgemeine Lösung der Wellengleichung für eine Störung, die sich in der bewegt X Richtung ist E ( R , T ) = E 0 ( X C T , j , z ) . Also nehme ich an

E ( R , T ) = 1 4 π ϵ 0 Q [ ( X C T ) 2 + j 2 + z 2 ] R ^
(Wo R ^ Punkte von der Ladung entfernt) ist eine Lösung der Wellengleichung.

Offensichtlich ist dies nur für nichtrelativistische Geschwindigkeiten gut. Und meine Notation für den Einheitsvektor ist nicht sehr gut ... Ich hoffe, es macht den Punkt.

Kommentar

Ich sehe, dass Sie Ihre Frage so bearbeitet haben, dass sie einen expliziten Ausdruck des relativistischen Ausdrucks für das Feld enthält. Ich denke, es ist klar, dass das, was ich für den nicht-relativistischen Fall getan habe, genauso gut für den relativistischen Fall funktioniert.

Ich habe diesen Kommentar gelöscht und meiner Antwort hinzugefügt.
Ich habe die Zeitabhängigkeit explizit nach Ihren Vorschlägen hinzugefügt, hoffe, es sieht richtig aus. Ich denke, der relativistische Teil kann der Einfachheit halber ignoriert werden @ Mathaholic
Es ist fast richtig ... Sie definieren nicht R ^ , und ich auch nicht. Mit einer korrekten Definition von R ^ , ich denke, was Sie geschrieben haben, ist offensichtlich eine Lösung der Wellengleichung!
Das Problem ist, ich habe v anstatt C in meiner Gleichung, also sehe ich nicht, wie es Maxwells Gleichung löst C drin. Können Sie ein wenig näher darauf eingehen?
Aha. Ich wusste, dass es zu einfach aussah. :)